Methods of Reasoning Questions - - Question 30

Certain items those with that hard to define quality called exclusivity have the odd property, when they become avail...

Alex07 September 30, 2013

Help

What does the answer choice mean?

Reply
Create a free account to read and take part in forum discussions.

Already have an account? log in

Naz October 1, 2013

The stimulus tells us that selling exclusive items by asking too low a price is a mistake because it "calls into question" the exclusivity of the item, which is their "chief appeal." Therefore, the argument concludes that a pricing model of "too high" is a better selling strategy for these exclusive items.

(A) is the CORRECT answer because it delineates how the "rejected alternative," pricing the exclusive items too low, is actually "counterproductive" because it calls into question the quality that makes the item appealing in the first place - being exclusive. Therefore, the recommended pricing strategy is preferred since it lacks this "counterproductive feature."

(B) is incorrect because the advantages of the rejected alternative were not discussed, and therefore we do not know whether they have their advantages in common and if this is why the argument recommends this specific pricing strategy.

(C) is incorrect because the topic of "experience" was never even broached in the stimulus and therefore we cannot say that is why the argument recommends this specific pricing strategy.

(D) is incorrect because we do not know whether the recommended strategy relies on prospective buyers' estimates of value since it was never discussed in the stimulus. Further, even if the strategy did not rely on prospective buyers' estimates of value, this is not why the argument recommends this pricing strategy. The argument recommends it because this strategy does not take away from the main appeal of the items, which is their exclusivity.

(E) is incorrect because we do not know whether or not "the error" of "setting the price too high" will likely go unnoticed. Therefore, we cannot say that the argument recommends the strategy for this reason.

Hope that was helpful! Let me know if you have any more questions.